#author("2022-06-18T09:40:46+09:00","","")
#mathjax

*「よくわかる解析力学」(東京図書)サポート掲示板 [#f859e7cb]

[[よくわかる解析力学サポートページに戻る>http://irobutsu.a.la9.jp/mybook/ykwkrAM/]]

-[[mathjax>http://www.mathjax.org/]]を使って、TeX形式で数式を打てるようにしてあります。$または$$(もちろんほんとは全角じゃなく半角の「ドル」です)で囲んで入力してください。
-spam避けに、httpを含む文章と、英字のみの文章は登録できなくしてあります。

&color(Red){「サポート掲示板3」が不具合で書き込めなくなりましたので作りました。こちらに書き込んでください。};

#article
**p224の(q,p+δp)のδtにおける位置の変化について [#z9054705]
>[[Jun]] (2022-06-18 (土) 09:40:46)~
~
p+δpのδtの変化ですが、本ではハミルトニアン(δpδtが掛かっている方)の偏微分がqの後pになっていますが、pの後qではないでしょうか。mathjaxの使い方が分からずこのような質問の書き方になりましたが、お願いします。~

//

#comment

**第4章 [#z9c7f605]
>[[SH]] (2022-05-25 (水) 10:06:41)~
~
P91のFAQの下の行 ~
-grad(なにか)の「なにか」を求めるのに、なぜ「なにを変分したら-md²x/dt²(t)・δx(t)が出てくるか」を考えるのですか?~
~
最終的なS試({x(*)})は(4.19)だと思うのですが、(4.18)でLとU(x)が出てくるのはなぜでしょうか。~

//
- gradは微分。微分と変分は言葉は違いますが、「ある量を変化させたときにその量に依存する量がどう変化するか」を求めているという意味では、同じ計算です。 -- [[前野]] &new{2022-05-25 (水) 12:06:08};
- (4.18)の時間積分が(4.19)、逆にいえば求めたS試である(4.19)は(4.18)という量の時間積分になっているということを示してます。 -- [[前野]] &new{2022-05-25 (水) 12:07:02};
- (1)出てくるようにしたいのが『-md²x/dt²(t)・δx(t)』である理由はなんですか。(2)(4.18)の「直前の結果を整理すると」について、p91-93で運動エネルギーにしか触れてないのに、急にLやU(x)が出てくる理由は何でしょうか。 -- [[SH]] &new{2022-05-25 (水) 16:02:41};
- ああ、Uが出てきたことに関する質問だったんですね。Uに関しては普通の微分で${\mathrm dU\over \mathrm dx}$が出てくるのは(4.5)のあたりでわかっていたので、これにさらに$m{\mathrm d^2 x\over\mathrm dt^2}$も出てくるようにするにはどうすればいいのかな、 -- [[前野]] &new{2022-05-25 (水) 17:56:08};
- (4.6)から後の話です。 -- [[前野]] &new{2022-05-25 (水) 17:56:24};
- P93の(4.19)から(4.20)が導かれる計算過程を教えていただきたいです。 -- [[SH]] &new{2022-05-26 (木) 12:45:47};
- オイラー・ラグランジュ方程式そのまんまです。 -- [[前野]] &new{2022-05-26 (木) 13:37:46};
- 理解できました。ありがとうございました。 -- [[SH]] &new{2022-05-27 (金) 10:18:45};

#comment

**微分の計算について [#s2aa294b]
>[[大学一年生]] (2022-03-29 (火) 03:09:55)~
~
巻末の数学的知識の解説のお陰もあり本書はあまり問題無く読み進められているのですが、もっと初歩的な部分で自分の理解が足りていないと感じたのでこちらにご質問させていただきます。~
~
$f(x(t), t) =X(x(t)) + T(t)$ とするとき、~
~
$$\displaystyle\frac{\partial f}{\partial x} = \displaystyle\frac{\partial X}{\partial x}, \, \displaystyle\frac{\partial f}{\partial t} = \displaystyle\frac{\partial T}{\partial t},$$~
~
$$\displaystyle\frac{\mathrm{d} f}{\mathrm{d} t} = \displaystyle\frac{\partial X}{\partial x} \, \displaystyle\frac{\mathrm{d} x}{\mathrm{d} t} + \displaystyle\frac{\partial T}{\partial t}$$~
~
であり、$\displaystyle\frac{\mathrm{d} f}{\mathrm{d} x}$ というものない(そのような量を考えることはない)、という理解は合っていますでしょうか。~
~
同様に、$g(x(t), t) =X(x(t)) \, T(t)$ であった場合には ~
~
$$\displaystyle\frac{\partial g}{\partial x} =\displaystyle\frac{\partial X}{\partial x} \, T(t) = \displaystyle\frac{\mathrm{d} X}{\mathrm{d} x} \, T(t),$$~
~
$$\displaystyle\frac{\partial g}{\partial t} =X(x(t)) \, \displaystyle\frac{\partial T}{\partial t} =X(x(t)) \, \displaystyle\frac{\mathrm{d} T}{\mathrm{d} t},$$~
~
$$\displaystyle\frac{\mathrm{d} g}{\mathrm{d} t} =\displaystyle\frac{\mathrm{d} X}{\mathrm{d} t} \,T(x) +X(x(t)) \, \displaystyle\frac{\mathrm{d} T}{\mathrm{d} t} =\displaystyle\frac{\partial X}{\partial x} \, \displaystyle\frac{\mathrm{d} x}{\mathrm{d} t} \, T(t) +X(x(t)) \, \displaystyle\frac{\mathrm{d} T}{\mathrm{d} t}$$~
~
であり、$\displaystyle\frac{\mathrm{d} g}{\mathrm{d} x}$ というものない(そのような量を考えることはない)ということでしょうか。~
~
あるいは、$t \mapsto x(t)$ の逆写像 $x \mapsto t(x)$ を考えて~
~
$$\displaystyle\frac{\mathrm{d} f}{\mathrm{d} x} = \displaystyle\frac{\mathrm{d} X}{\mathrm{d} x} + \displaystyle\frac{\mathrm{d} T}{\mathrm{d} t} \, \frac{\mathrm{d} t}{\mathrm{d} x},$$~
~
$$\displaystyle\frac{\mathrm{d} g}{\mathrm{d} x} = \displaystyle\frac{\mathrm{d} X}{\mathrm{d} x} \, T(t) +X(x(t)) \, \frac{\mathrm{d} T}{\mathrm{d} t} \, \frac{\mathrm{d} t}{\mathrm{d} x}$$~
~
のようにするのでしょうか。~
~
また、これまで何冊か大学数学や物理の本を読んできて偏微分は(偏微分する変数以外を定数扱いするため例えば $X(x(t))$ を $t$ で偏微分するときに $x$ は $t$ の式として展開せずに「定数」 $x$ として扱うように)「中身の依存関係まで見に行かない」微分、(全)微分は(例えば $X(x(t))$ を $t$ で全微分するときに $x$ を $t$ の式として展開して微分したように)「中身の依存関係まで見に行く微分」というイメージを抱いているのですが、このような認識を持っていても大丈夫でしょうか。~
~
このイメージのもとでは、上の例では($t$ は $x$ に依存しない量であるため、$t$ を $x$ の式として展開できない、すなわち「中身まで見に行けない」ため)$\displaystyle\frac{\mathrm{d} f}{\mathrm{d} x}$ や $\displaystyle\frac{\mathrm{d} T}{\mathrm{d} x}$ などのような量は考えられないだろうと思いました。~
~
初歩的かつ長い質問で申し訳ありませんが、ご回答いただけると大変幸いに存じます。よろしくお願いいたします。~

//
- すみません。本文が長すぎたのかシステムのバグを踏んでしまったのか、(少なくともこちらの環境では)本文が正常に表示されていません。正しく表示された本文のスクリーンショットをこちらにアップロードいたしましたのでこちらで同じ内容をご確認いただけます(p の前の @ を外してください)。 htt@ps://imgur.com/GZhncrR -- [[大学一年生]] &new{2022-03-29 (火) 03:18:05};
- 最初に書いた $f$ の(偏/全)微分に含まれる $\displaystyle\frac{\partial X}{\partial x}$ や $\displaystyle\frac{\partial T}{\partial t}$ はそれぞれ $\displaystyle\frac{\mathrm{d} X}{\mathrm{d} x}, \, \displaystyle\frac{\mathrm{d} T}{\mathrm{d} t}$ のように書けます(直し忘れました)。 -- [[大学一年生]] &new{2022-03-29 (火) 03:59:09};
- まず厳密な立場で考えると、$f(x(t),t)$と書いたならそれはもう「$t$の関数」であって「$x,t$の関数」ではありませんから、「${\partial f\over\partial x}$を考えるのは無意味」です。${\partial f\over\partial x}$を考えるのは「$x=x(t)$を代入する前」の段階でのみです。 -- [[前野]] &new{2022-03-29 (火) 05:34:32};
- そういう意味で、この状況では「${\mathrm df\over\mathrm dx}$もなければ、${\partial f\over\partial x}$もない」というのが正しいです。 -- [[前野]] &new{2022-03-29 (火) 05:35:53};
- ただし、多くの場合「いったん$x$を$t$の関数ではないと考えて、計算終わったあとで$x=x(t)$を代入する」という操作を「暗黙のうちに」やっている場合が多々あります。その場合、「計算途中」として${\partial f\over\partial x}$が出現するわけです。 -- [[前野]] &new{2022-03-29 (火) 05:37:30};
- 偏微分は、「何と何を変数にしているか」を変えれば違う結果を出す計算なので、計算の途中で常に「今変数としているのはなにか?」を意識してないといけません。そういう意味で、$f(x(t),t)$と書いたら変数は$t$なので「$x$で微分(全微分も偏微分も)」はないわけです。ただ計算の途中経過として$f(x,t)$を考えることはあるので、あくまでその途中経過の中で「$x$で偏微分」が登場します。 -- [[前野]] &new{2022-03-29 (火) 05:40:35};
- なお、$f(x,t(x)$と書ける場合はそう書いてもいいけど、その場合は「$x$の関数」であって、「$t$で微分」はないです。しかし計算の途中経過では出てくることもある。 -- [[前野]] &new{2022-03-29 (火) 05:41:50};
- (記法の問題で誤解が生じやすいけれど)数学的には $f(x(t), t)$ という記法はまず $\mathbb{R}^2 \to \mathbb{R}$ の関数 $f(x, t)$ を考えて、後から $x$ と $t$ の関係を持ち出してきて二変数の定義域を $\mathbb{R}^2$ から曲線(変数が増えれば曲面など)$x = x(t)$ に制限している(そのため $L(x(t), \dot{x}(t))$ に対して定義域を $\mathbb{R}^n$ から制限しない段階で $\displaystyle\frac{\partial L}{\partial x}$ などを考えることができる)と考えてよろしいでしょうか。 -- [[大学一年生]] &new{2022-03-29 (火) 06:32:15};
- また、本来はきちんと明記するべきですが $\displaystyle\frac{\partial f}{\partial x}, \, \displaystyle\frac{\partial f}{\partial t}$ とあったら $x$ と $t$ の関係を代入する前の二変数関数 $f(x, t)$ について考えていて、$\displaystyle\frac{\mathrm{d} f}{\mathrm{d} t}$ とあったら $x = x(t)$ を代入して $t$ のみの関数になった $f(x(t), t)$(更に一変数関数らしく書けば $F(t) \equiv f(x(t), t)$ など)を考えていて、(あまり見ないかもしれないけど)$\displaystyle\frac{\mathrm{d} f}{\mathrm{d} x}$ とあったら $t = t(x)$ という逆対応を代入して $x$ のみの関数になった $f(x, t(x)) \ (\equiv G(x))$ を考えていると思えば誤解は無さそうだというわけですね。 -- [[大学一年生]] &new{2022-03-29 (火) 06:32:42};
- そして($x \mapsto t(x)$ の逆対応が無いなどで)$f(x, t)$ を $x$ のみの関数にできない場合には $\displaystyle\frac{\mathrm{d} f}{\mathrm{d} x}$ をそもそも考えない(ただし偏微分の方は普通に二変数関数のままにして考えられる)ということですね。同じことの復唱のような感じで申し訳ありませんが、このような理解で問題ありませんでしょうか。悩んでいたことが晴れてすっきりしております。ありがとうございます。 -- [[大学一年生]] &new{2022-03-29 (火) 06:33:33};
- 「まず〜〜〜を考えて、後から〜」の部分は、別に後から気づくわけではなく$x$が$x(t)$なのは最初からわかっている場合もあるので必ずそうだとは限りませんが、理解としてはそれでいいと思います。また、本来はきちんと明記すべきところをさぼって省略してしまうことが多いのはおっしゃるとおりです。 -- [[前野]] &new{2022-03-29 (火) 07:09:14};
- 「まず」「後で」は実際にその間に時間差があるということではなく、比喩的に用いた表現でした。思考が整理されてとても感謝しております。ご回答ありがとうございました。 -- [[大学一年生]] &new{2022-03-29 (火) 07:19:41};

#comment

**p.36~p39の変分について [#u8db1430]
>[[RR]] (2022-03-25 (金) 23:42:25)~
~
1. 関数$y$に対する変分$\delta y$と, 汎関数$I$に対する変分$\delta I$の違いについて触れられていませんが, これらは異なる概念と考えて良いでしょうか?
また, この場合$\delta I$に対する$\delta y$は$\frac{\mathrm dy}{\mathrm dx}$に対する$\mathrm dx$のようなものと考えて良いでしょうか?~
~
2. 異なるとして前者についての質問です. ~
導関数の変分$\delta\frac{\mathrm dy}{\mathrm dx}$がなぜ元の関数とその変分の和$(y+\delta y)$と元の関数$y$のそれぞれの微分の差になるのでしょうか?~
変分$\delta y$の元の関数$y$に対する依存が分かりませんでした. 有限次元ベクトル$\mathbf x$の関数$f(\mathbf x)$の場合, 微小ベクトル$\mathrm d\mathbf x$は$x$とは関係なかったと思います. ~
これと同様に変分$\delta y$も関数空間上で何らかの意味で微小な関数であって, 元の関数$y$とは関係ないと考えた方が自然なように感じます.~
また, $\delta y$を任意の微小関数していることからも, $y$の関数形に依存しない方が良いのではないでしょうか?~
~
3. p.38で$X=\delta y^\prime$とするのであれば, (2.39)式の第二項の$\delta y$は$\delta y^\prime$ではないでしょうか? ~

この場合, (2.41)式の$\delta y$は$\delta y^\prime$になるので, 微分と交換して$\delta y^\prime=(\delta y)^\prime$となります. 
直後のFAQの通り考えると, これでは係数=0とできないのではないでしょうか?~

//
- 3について, $\frac{\mathrm d(\delta y)}{\mathrm d x}$となっていることを見落としていました。この点に関しては解決したため, 1, 2についてご教授いただきたいです.  -- [[RR]] &new{2022-03-26 (土) 00:16:50};
- 1.について。関数$y$と汎関数$I$は違うもの($y$は$x$の関数、$I$は$y$という関数の関数)なので、当然二つは違うものです。ただし、$I(y(x_1),y(x_2),y(x_3),\cdots,y(x_N))$のように、汎関数を「関数$y(x)$の各点各点での値を変数に持つ関数」と捉えるなら、「素直な拡張」だと言えるでしょう。 -- [[前野]] &new{2022-03-26 (土) 08:33:07};
- $f(x,y,z)$のような多変数関数を考えて、$\delta f(x,y,z)={\partial f\over\partial x}\delta x +{\partial f\over\partial y}\delta y +{\partial f\over\partial z}\delta z$とやるように、$\delta I=\int {\partial I\over \partial y(x)}\delta y(x)$のように考えているという意味では、$\delta y$は$\delta x$のようなもの、という考えはそれでOKです。 -- [[前野]] &new{2022-03-26 (土) 08:35:52};
- 2.についてですが、まず、$y$は$x$の関数(省略せずに書けば$y(x)$)なので、$\delta y$も$\delta y(x)$です。$x$の関数であって、$y$の関数ではありません。 -- [[前野]] &new{2022-03-26 (土) 08:37:13};
- 導関数の変分がなぜ変分の微分になるのかは本で説明している通りで、これも省略せずに書くと導関数は変分前は${y(x+\Delta x)-y(x)\over \Delta x}$の極限で、変分後は${y(x+\Delta x)+\delta y(x+\Delta x)-y(x)-\delta y(x)\over \Delta x}$の極限になるからです。 -- [[前野]] &new{2022-03-26 (土) 08:40:43};
- 上でも述べましたが、$\delta y$は$y$の関数ではなく、$x$のみの関数です。 -- [[前野]] &new{2022-03-26 (土) 08:42:55};
- $\delta(y^\prime)$は暗に$y$を$y+\delta y$にした時の$y^\prime$の変化を表すことを意味しているのでしょうか? -- [[RR]] &new{2022-03-29 (火) 02:39:35};
- 「暗に」ではなく、δ(なんとか)の意味は「関数を変形する前後の(なんとか)の差」なのですから、まさにそういう意味です。 -- [[前野]] &new{2022-03-29 (火) 05:30:39};
- $\delta (y^\prime)$の変分を$y$を$y+\delta y$にした時の$y^\prime$の変化は, 「関数を変形する前後の差」ではなく, 「原始関数を変形させる前後の関数の差」ではないでしょうか? -- [[RR]] &new{2022-03-29 (火) 06:41:58};
- $\delta(y')$は「$y\to y+\delta y$と変化させたときの$y'$の変化」が定義です。それは「原始関数を変形させる前後の微係数の差」ということになると思います。それが「$\delta y$の微分」と同じになる、ということは本の中で説明しているとおりです。 -- [[前野]] &new{2022-03-29 (火) 07:04:39};

#comment

**p.339 (C.23) について [#w30351e3]
>[[大学生]] (2022-03-25 (金) 11:04:55)~
~
ここは直交曲線座標に限った話なのでしょうか。一般の座標系では $i \neq j$ のときに内積が $0$ にならないのではないかと思ってしまいました。~

//
- いいえ、一般の座標系です。$i\neq j$で内積が0になるように定義したのが「双対なベクトル」です(添字が上付きと下付きは同じベクトルではなく、この関係が成り立つように作ります)。 -- [[前野]] &new{2022-03-25 (金) 11:36:52};
- ありがとうございます。大きな勘違いをしていたことに気づきました。この辺の知識に関しては更に自習をして理解したいと思います。さっき少し調べただけの付け焼き刃な知識で -- [[大学生]] &new{2022-03-25 (金) 12:32:06};
- 申し訳ありませんが、 ja.wikipedia.org/wiki/%E5%8F%8C%E5%AF%BE%E5%9F%BA%E5%BA%95#%E4%BE%8B の「三次元ユークリッド空間において、基底 {e1, e2, e3} が与えられたとき、その二重直交(双対)基底 {e1, e2, e3} は次の式によって得ることが出来る」と書いてあるところの式のように具体的には定義されるようですね。 -- [[大学生]] &new{2022-03-25 (金) 12:34:59};
- 取りあえず、一般の座標系での話をしているということは理解できました。ありがとうございます。 -- [[大学生]] &new{2022-03-25 (金) 12:36:20};

#comment

**誤記? [#r0840273]
>[[冬]] (2022-03-22 (火) 12:26:32)~
~
細かいですが p.148 の第一段落に二箇所ある「作用」は「ラグランジアン」と、p.152 の (6.54) の直前の文の「〜の 2 次以上を無視した作用である。」は「〜の 3 次以上を無視したラグランジアンである。」とされたほうが正確な気がします。~

//
- ありがとうございます。確かにここはラグランジアンです。 -- [[前野]] &new{2022-03-23 (水) 08:31:21};
- あと、次数も3以上です(運動方程式だと2次以上無視なので、そっちと勘違いして書いてしまったようです)。 -- [[前野]] &new{2022-03-23 (水) 08:32:09};

#comment

**p.136 の (5.96) について [#cfe32856]
>[[大学生]] (2022-03-18 (金) 15:49:07)~
~
式 (5.96) は $G_j(\{q_*\}, \{Q_{\star}(\{q_*\})\}) = 0$ を微分して~
~
$\mathrm{d}G_j = \sum_{i=1}^{N-M} \frac{\partial G_j}{\partial q_i} \mathrm{d} q_i + \sum_{k=1}^{M} \frac{\partial G_j}{\partial Q_k} \mathrm{d} Q_k$~
~
$= \sum_{i=1}^{N-M} \frac{\partial G_j}{\partial q_i} \mathrm{d} q_i + \sum_{k=1}^{M} \frac{\partial G_j}{\partial Q_k} \sum_{i=1}^{N-M} \frac{\partial Q_k}{\partial q_i} \mathrm{d} q_i$~
~
$= \sum_{i=1}^{N-M} \left( \frac{\partial G_j}{\partial q_i} + \sum_{k=1}^{M} \frac{\partial G_j}{\partial Q_k} \frac{\partial Q_k}{\partial q_i} \right) \mathrm{d} q_i = 0$~
~
より成り立つと考えたのですが、正しいでしょうか。~

//
- そういう計算です。 -- [[前野]] &new{2022-03-19 (土) 02:55:55};
- ありがとうございます。 -- [[大学生]] &new{2022-03-19 (土) 03:16:10};

#comment

**p163の図について [#ifc815e0]
>[[み]] (2022-03-14 (月) 22:16:27)~
~
p163の図について質問があります。~
pが腹の数になるのはなぜでしょうか?また、振幅がCsin(pπ/ℓ)xと分かるのはなぜでしょうか?~

//
- これは(6.66)→(6.70)と少しずつ連続な話にしていった結果なので、pが節の数なのも振幅がこうなのも(6.66)、(6.70)と同様です。 -- [[前野]] &new{2022-03-15 (火) 04:25:00};
- モードがsinで表せる→一般的に6.70でかける縦波のsinカーブができる→pが大きいほど振動が多く腹の数が増える→p=1が最も振動数が少なく腹が1つで、その後も腹の数がp個になる→振幅は6.70から√(2/N+1) -- [[み]] &new{2022-03-15 (火) 13:03:42};
- すみません、書き途中です --  &new{2022-03-15 (火) 13:06:02};
- 振幅は6.70から√(2/N+1)となるところまでは分かりましたがそこからが分かりません。 --  &new{2022-03-15 (火) 13:34:33};
- そこまでわかったら、後は何がわからないのが問題なのでしょう?? この段階の答えとしては$C_p \sin {p\pi\over \ell}x$で角振動数${p\pi\over \ell}\sqrt{{\kappa\over\rho}}$で振動ということで、わかることは全部わかったと思いますが(角振動数の部分についてですか?)  -- [[前野]] &new{2022-03-16 (水) 03:39:40};

#comment

**p153の 6.60について [#h7b570b5]
>[[み]] (2022-03-13 (日) 00:11:58)~
~
θ1:θ2=√m:√Mについてですが、この関係が固有ベクトルに対応したものだということは以前の掲示板を確認して分かったのですが、なぜ対応しているのか具体的な式、考え方が知りたいです。よろしくお願い致します。~

//
- (6.58)を解いた結果が二つの固有ベクトルなわけですが、(6.58)は(6.55)という固有値方程式(通常とはちょっと違う形だけど、これも固有値方程式のようなものだ、というのは151ページのあたりに書いてあります)を解いた結果のベクトルなので、結果の$\vec v$はその意味での固有ベクトルです。 -- [[前野]] &new{2022-03-13 (日) 09:05:19};
- ありがとうございます。そこまでは理解できましたが、その固有ベクトルからなぜθ1:θ2=√m:√Mがわかるののでしょうか? -- [[み]] &new{2022-03-13 (日) 14:04:01};
- $\left(\begin{array}{c}v_1\\ v_2\end{array}\right)$が$\left(\begin{array}{c}\sqrt{m}\\ \sqrt{M}\end{array}\right)$であるところまではわかっているでしょうか? -- [[前野]] &new{2022-03-13 (日) 14:10:25};
- 正確にいうと、$\left(\begin{array}{c}v_1\\ v_2\end{array}\right)$が二つの固有ベクトル$\left(\begin{array}{c}\sqrt{m}\\\sqrt{M}\end{array}\right)$と$\left(\begin{array}{c}-\sqrt{m}\\ \sqrt{M}\end{array}\right)$の線形結合になってます。そして、$\left(\begin{array}{c}v_1\\ v_2\end{array}\right)$は$\left(\begin{array}{c}\theta_1\\ \theta_2\end{array}\right)$です。 -- [[前野]] &new{2022-03-13 (日) 14:13:58};
- 固有ベクトルが線形結合になることまでは分かりますが、最後の(v1 v2)が(θ1 θ2)になるのが分かりませんでした。 -- [[み]] &new{2022-03-13 (日) 15:29:52};
- もともと、(6.53)に出てきた$\theta,\dot\theta$で書かれたラグランジアンがあるときの計算をしてますから、vがθになるのは「もともとθだったところをvに置き換えて計算したんだから、元に戻した」ということです。 -- [[前野]] &new{2022-03-13 (日) 15:36:02};
- (6.43)のようなラグランジアン(これは$x_i,\dot x_i$で書かれている)があるとき、(6.44)のような式を解いて「固有ベクトル」を求めるとその固有ベクトルで$x,\dot x$がわかる・・・というのが一つ前の節のストーリーで、この6.2.4節も同じストーリーに乗っかります。 -- [[前野]] &new{2022-03-13 (日) 15:38:22};
- というわけで、6.2.3節でやっていることをもう一度復習して「それと同じことをやっている」と思いながら式を追っかけてみるとわかるんじゃないかと思います。 -- [[前野]] &new{2022-03-13 (日) 15:39:06};
- P148の計算では、固有ベクトルTを単に座標変換後のラグランジアンを簡単にするために使っていたものだとしか考えていませんでしたが、このTも座標xと対応していると考えることができて、今回もそれと同じことをしているという認識で大丈夫でしょうか? -- [[み]] &new{2022-03-13 (日) 17:09:42};
- このTは、もちろん座標ですよ。 -- [[前野]] &new{2022-03-13 (日) 17:16:58};
- x1=T1X1+T2X2のようにしてxからXにしたいという認識であっていますか?このTはただの係数だと思っていましたが座標とはどうゆうことでしょうか? -- [[み]] &new{2022-03-13 (日) 18:19:32};
- xとT(vとθも)文字が変わっているだけで同じものです。 -- [[前野]] &new{2022-03-13 (日) 18:58:19};
- (6.43)の$\left(\begin{array}{c}x_1\\ x_2\end{array}\right)$が$\left(\begin{array}{c}(\vec T_1)_1\\  (\vec T_1)_2\end{array}\right)$である場合と$\left(\begin{array}{c}(\vec T_2)_1\\  (\vec T_2)_2\end{array}\right)$になる場合を考えましょう、というのが(6.45)です。つまりはxの特殊な例として$\vec T_1,\vec T_2$を持ってきています。今考えている方程式は線形なので、二つ解を見つけたらその線形結合$a_1\vec T_1+a_2\vec T_2$も解です。 -- [[前野]] &new{2022-03-13 (日) 19:04:11};
- どうように、θの特殊な例としてvを求めようとしているのが(6.55)です。 -- [[前野]] &new{2022-03-13 (日) 19:05:57};
- なるほど、計算の過程でxとTが結びつくのではなくて、最初からxのことをTとおいて計算しているということですか! -- [[み]] &new{2022-03-13 (日) 19:21:42};
- θ1、θ2の特殊な場合として√m、√Mが求まり、固有ベクトルが線形結合でかけるので比としてθ1:θ2=√m:√Mが結論できると理解できました。 -- [[み]] &new{2022-03-13 (日) 19:34:09};
- 丁寧にご対応頂きありがとうございます。 -- [[み]] &new{2022-03-13 (日) 19:36:13};

#comment

**p.88 から p.89 にかけて [#r5c1132e]
>[[大学生]] (2022-03-12 (土) 18:43:23)~
~
p.88 までに書かれている内容と、ラグランジアン($\frac{1}{2} m |\dot{x}|^2 - V(x)$)を導入すると p.89 以降の議論によりラグランジアンの時間積分が停留値をとるという条件から運動方程式が再現することは(それぞれ独立に)理解したつもりなのですが、p.88 の下に書かれている「無限個の連立方程式」を導く手段としてオイラー・ラグランジュ方程式が出てくることが理解できておらず、p.88 の内容と p.89 からの内容が頭の中でうまく繋がっていません。~
アドバイスをいただければ幸いに存じます。~

//
- まず作用が(4.11)にあるような「すべての$\vec x(t)$の関数」で、それに対する変分が0になるという式が「無限個の連立方程式』になるという点はわかっているでしょうか? -- [[前野]] &new{2022-03-13 (日) 08:58:11};
- オイラー/ラグランジュ方程式というのは結局、(4.11)の変分が0というのをまとめて言っているだけのことです。 -- [[前野]] &new{2022-03-13 (日) 09:00:06};
- なるほど、変分が 0 になる、すなわち $\delta S = S(\{ \vec{x}_{*} + \delta \vec{x}_{*} \}) - S(\{ \vec{x}_{*} \}) = 0$ の条件が、$\vec{x}_{i}$ のみを動かして他の変数を固定したときの $S(\vec{x}_{1}, \dots, \vec{x}_{i-1}, \vec{x}_{i} + \delta \vec{x}_{i}, \vec{x}_{i+1}, \dots, \vec{x}_{N}) - S(\vec{x}_{1}, \dots, \vec{x}_{i-1}, \vec{x}_{i}, \vec{x}_{i+1}, \dots, \vec{x}_{N}) = 0$ の条件を包含しているので $\vec{x}_{i}$ での偏微分が 0 となる「無限個の連立方程式」が導かれるということですね。当たり前なのにあまり意識していませんでした。  また、式 (4.8) の「なにか」に入る量が作用であることも理解していませんでした。  納得できました。ありがとうございます。 -- [[大学生]] &new{2022-03-14 (月) 08:21:58};

#comment

**p.72 ポテンシャルの安定点について [#a7e8d29c]
>[[冬]] (2022-03-08 (火) 16:01:33)~
~
$\displaystyle\frac{\partial U}{\partial r} = \displaystyle\frac{\partial}{\partial r} \left( \displaystyle\frac{1}{4} Kr^4 - \displaystyle\frac{1}{2} kr^2 \right) = Kr^3 - kr$ より停留点は $r = 0$ または $r^2 = k/K$ を満たす点であり、$\displaystyle\frac{\partial^2 U}{\partial r^2} = 3Kr^2 - k$ より $\left.\displaystyle\frac{\partial^2 U}{\partial r^2} \right|_{r = 0} = -k, \ \left.\displaystyle\frac{\partial^2 U}{\partial r^2} \right|_{r^2 = k/K} = 2k$ となり、$k > 0$ ならばこれらの停留点のうち $r^2 = k/K$ を満たす点が特に安定点となると解釈したのですが、合っていますでしょうか。~

//
- つまり、$k < 0$(その場合 $K < 0$ でないと $r^2 = k/K$ を満たす点はなくなりますが)の状況では $r = 0$ の原点が唯一の安定点となるが、ここでは $k > 0$ と考えているということでよろしいでしょうか。例えばバネ定数の $k$ は正なので、ここでもそのように正の $k$ を考えているのだと思いますが念のため質問をさせていただきました。 -- [[冬]] &new{2022-03-08 (火) 16:08:57};
- ああ確かに、ここではkやKが正とも負とも書いてないですね。どっちも正の場合を書いてます。 -- [[前野]] &new{2022-03-08 (火) 16:12:42};
- 回答について、了解しました。ありがとうございます。 -- [[冬]] &new{2022-03-08 (火) 17:05:29};

#comment

**p132 式5.81について [#l56226e8]
>[[大学1年]] (2022-02-09 (水) 13:57:15)~
~
拘束Gのあるオイラーラグランジュ方程式について質問があります。~
∂G/∂xの比例係数λが時間の関数として導入されていますが、時間だけでなくxにも依存して良いと考えたのですがどうでしょうか。~
宜しくお願い致します。~

//
- 導入の時点ではx(t)やy(t)と並び立つ「力学変数の仲間」として導入するので、最初の段階ではλ(t)なのです。このあとλに関するオイラー/ラグランジュ方程式を出したりして、その方程式を解いた結果としてλがxやyを含む関数になることはアリです。 -- [[前野]] &new{2022-02-09 (水) 15:42:47};
- 納得出来ました。ありがとうございます。 -- [[大学1年]] &new{2022-02-09 (水) 16:56:28};

#comment

**p108 (4.76)(4.77)について [#m98782da]
>[[Sato]] (2021-12-11 (土) 14:48:50)~
~
p108の(4.76),(4.77)のmに添え字は必要ないのでしょうか.~
ご教授いただければ幸いです.~

//
- すいません、書き忘れてます。$\ddot x_1$の前は$m_1$で、$\ddot x_2$の前は$m_2$です。 -- [[前野]] &new{2021-12-11 (土) 14:59:42};
- 返信ありがとうございます. --  &new{2021-12-11 (土) 15:01:49};

#comment

**無題 [#xa35aab5]
>[[大学1年]] (2021-12-02 (木) 21:20:25)~
~
p77の式3.62から式3.63の詳しい導出過程が知りたいです。宜しくお願いします。~

//
- 書いてあるとおりのことしかしてません。第1項なら${1\over y^2}$ですから${y'\over y}$を掛けると${y'\over y^3}$。これを積分すれば$-{1\over 2y^2}$です(逆に$-{1\over 2y^2}$を微分すれば${y'\over y^3}$)。 -- [[前野]] &new{2021-12-03 (金) 14:07:47};
- 第2項も、${y'\over y}{d\over dx}\left({y'\over y)}\right)$になるので積分すると${1\over2}\left({y'\over y}\right)^2$(微分すればこれでいいことは確認できます)。ここの計算は、運動方程式でよくやる$v m{dv\over dt}$を積分して${1\over 2}mv^2$にするのと同じ理屈です。 -- [[前野]] &new{2021-12-03 (金) 14:09:56};
- 計算できました。ありがとうございます。 -- [[大学1年]] &new{2021-12-03 (金) 18:51:30};

#comment

**p.152 二重振り子(6.52)式の近似 [#y151fa31]
>[[ぺろり]] (2021-10-20 (水) 12:03:38)~
~
(6.52)から(6.53)の変形は、テイラー展開で2次の項まで残しているのは分かるのですが、$\cos (\theta _1-\theta _2)$を1と近似して良い理由がわかりません。2次の項まで残すのなら、$\cos (\theta _1-\theta _2)$から$\theta _1\theta _2$や$\theta _1^2$や$\theta _2 ^2$といった項も残ると思うのですが。~

//
- 問題の項は$\dot\theta_1\dot\theta_2\cos(\theta_1-\theta_2)$なので、$\dot\theta_1\dot\theta_2$の時点ですでに「2次の項」なのです。 -- [[前野]] &new{2021-10-21 (木) 17:33:01};
- 「微小振動」では$\theta <<1$ -- [[ぺろり]] &new{2021-10-22 (金) 00:59:16};
- 「微小振動」では$\theta <<1$とみなすのに加え、その時間微分について$\dot{\theta}<<1$も前提であるということですか? -- [[ぺろり]] &new{2021-10-22 (金) 01:00:59};
- そうですね、そう考えます。 -- [[前野]] &new{2021-10-22 (金) 08:34:42};
- ありがとうございます。 --  &new{2021-10-23 (土) 04:59:35};

#comment

**p.292 球対称ポテンシャル内の3次元運動 [#w11e6db4]
>[[林]] (2021-09-20 (月) 10:22:36)~
~
p.292の1.2行目で定数lは全角運動量Lに等しいと書いてありますが、なぜ等しくなるかが分かりません。5.2.3節から全角運動量を求めると  \[\ \vert \vec{L} \vert = {\sqrt{(\dot{\theta})^2+(\dot{\phi})^2\sin{{}^2\theta}}  }\]\ となりますが、これと定数lの関連性について教えていただきたいです~

//
- (11.43)まで戻ると、全エネルギーEが、第1項から順に、r方向の速度による運動エネルギー、θ方向の速度による運動エネルギー、φ方向の速度による運動エネルギー、位置エネルギーの4つの和になってます。第2項と第3項が「角運動量によるエネルギー」になっているので、その項を見れば$\ell$が角運動量になっていることがわかります。 -- [[前野]] &new{2021-09-20 (月) 10:32:58};
- ようやく理解できました。ありがとうございます --  &new{2021-09-20 (月) 14:45:13};

#comment

**P.287 調和振動子 [#g0575e09]
>[[林]] (2021-09-19 (日) 12:54:17)~
~
ハミルトニアンがtに依存しないので変数分離でき、保存量として~
Eが見つかり、それを新しい運動量Pとするとハミルトンの主関数において、~
独立変数をq,Pにとると新しい座標Qが(∂S ̅)/∂Pになるのはわかるのですが、なぜその新しい座標Qが保存すると言えるのでしょうか~

//
- ここでやっているハミルトン・ヤコビ方程式の流れというのは「正準変換することで新しいハミルトニアンを0にする」という計算です。ですから、新しい座標Qと新しい座標Pで表現したハミルトニアンが0なので、時間発展しません。 -- [[前野]] &new{2021-09-19 (日) 13:44:41};
- 新しいハミルトニアン0の式に代入すると確かに0になりますね。 --  &new{2021-09-19 (日) 16:25:09};
- Wを求めないといけないと思い込んでいました。理解しました、ありがとうございます --  &new{2021-09-19 (日) 16:27:18};
- すいません、もう一つ質問させてください --  &new{2021-09-19 (日) 16:39:43};
- 上の場合、新しい運動量PはS⁻に含まれる定数以外の保存量なら何を選んでもよいのでしょうか --  &new{2021-09-19 (日) 16:44:47};
- そのあたりの手順は285ページあたりに書いてあるとおりで、保存量となる数が見つかればそれが運動量になるように正準変換していけるわけです。「何を選んでもいい」と言われると自由度がありそうですが、実際にはむしろ難しい問題では定数が見つからないです(例として挙げているのは解ける問題なので見つかりますが)。 -- [[前野]] &new{2021-09-19 (日) 17:05:38};
- 納得しました。ありがとうございます --  &new{2021-09-19 (日) 17:49:57};

#comment

**p.110 演習問題4-2について [#g30b53bb]
>[[michi]] (2021-09-18 (土) 21:41:30)~
~
表題文中にある「重心の運動が自由粒子の運動と等価になる」とはどういうことかご教授いただけないでしょうか.~

//
- 「等価になる」というのは「ラグランジアンが同じ形」ということです。つまり、${1\over 2}M(\dot X)^2$のようになります。 -- [[前野]] &new{2021-09-19 (日) 08:13:58};
- 運動方程式が$M\ddot X=0$のようになる、と言っても同じです。 -- [[前野]] &new{2021-09-19 (日) 08:14:21};
- やはりよくわかりません.問題のヒントと回答に沿って教えてください. -- [[michi]] &new{2021-09-19 (日) 08:52:55};
- ヒントでは3つの質点個別の運動エネルギーとポテンシャルからラグランジアンを求めて,そこから書く質点ごとの運動方程式を求めていると思います. -- [[michi]] &new{2021-09-19 (日) 08:55:16};
- その後,回答に移ったときの「$\Sigma mi\ddot{x}i$が0になるために」がどこから出てきたものなのかがわかりません. -- [[michi]] &new{2021-09-19 (日) 08:59:51};
- ああ、ごめんなさい。解答の1行目の式に(つけたつもりだったのですが)${\mathrm d\over\mathrm dt}$が抜けてますね(古いバージョンではこうなってましたが、今アップロードされているバージョンでは$\ddot x$になってました)。 -- [[前野]] &new{2021-09-19 (日) 09:20:50};
- ヒントの式がどこから出てくるかとうと、まず、重心の「運動方程式」を考えてみると、${\mathrm d\over \mathrm dt}\left((m_1+m_2+m_3)\dot{\vec x_G}\right)=0$です(重心の運動は自由粒子なので)。 -- [[前野]] &new{2021-09-19 (日) 09:27:19};
- これは${\mathrm d\over \mathrm dt}\left(m_1\dot{\vec x_1}+m_2\dot{\vec x_2}+m_3\dot{\vec x_3}\right)=0$と同じです。 -- [[前野]] &new{2021-09-19 (日) 09:28:28};
- 問題で何を問われているのかがわかりました.ありがとうございました.加えて(E.77)式,(E.78)式について教えてください. -- [[michi]] &new{2021-09-19 (日) 09:59:33};
- (E.77)式は「各質点の位置で求めたポテンシャルの勾配ベクトルの総和が0ベクトルになる」という意味だと思いますが,ポテンシャルの並進不変性とどうつながるのかがわかりません. -- [[michi]] &new{2021-09-19 (日) 10:02:53};
- やはりよくわかりません.問題のヒントと回答に沿って教えてください. -- [[michi]] &new{2021-09-19 (日) 10:03:57};
- 微分というのはそもそも「xをちょっと変化させたときの変化」を計算するものです。(E.77)は、$x_1,x_2,x_3$を全部一斉に変化させたときの変化量を計算していることになります。これが0だというのは物理的にいうと、「$x_1,x_2,x_3$を同時に動かしても物理的内容は変わらない」ということです。 -- [[前野]] &new{2021-09-19 (日) 10:21:38};
- (E.78)がまさに「$x_1,x_2,x_3$を一斉に$\vec \epsilon$だけ動かしても、ポテンシャル$V$が変化しない」ということを表現した式になってます。これをテーラー展開して$\epsilon$の1次を取り出したと思えば、それは$\sum_i{\partial V\over\partial \vec x_i}=0$ということになります。 -- [[前野]] &new{2021-09-19 (日) 10:23:19};
- (E.78)は$\vec{\epsilon}$の位置変化に対して,各質点位置のポテンシャルの変化がそれぞれ0という式ですよね?(E.77)はあくまで「総和が0」という式なので必ずしも同じことを言っているとは思えないのですが... -- [[michi]] &new{2021-09-19 (日) 10:55:26};
- (E.78)は$V(\vec x_1+\vec\epsilon,\vec x_2+\vec\epsilon,\vec x_3+\vec\epsilon)-V(\vec x_1,\vec x_2,\vec x_3)$の省略形と読んでください。省略をしすぎたかもしれません。 -- [[前野]] &new{2021-09-19 (日) 10:58:04};
- 理解しました.ありがとうございました. -- [[michi]] &new{2021-09-19 (日) 12:19:11};

#comment

**問い10-11 [#l0327cc6]
>[[吉川晃生]] (2021-09-17 (金) 22:04:36)~
~$\frac{\partial P_i (q,p)}{\partial q_j} =  \frac{\partial p_j (Q,P)}{\partial Q_i}$ 
の導出過程を詳しく教えていただきたいです。~
- 解答に書いてあるのでは、どこが不満なのでしょう? -- [[前野]] &new{2021-09-17 (金) 22:11:23};
- (D.117)をまねて  $\frac{\partial P_i (q,Q(q,p))}{\partial q_j} =\frac{\partial P_i (q,Q)}{\partial q_j}+  \Sigma_k \frac{\partial P_i (q,Q)}{\partial Q_k}\frac{\partial Q_k (q,p)}{\partial q_j}= \frac{\partial P_i (q,Q)}{\partial q_j}+\Sigma_k \frac{\partial P_k (q,Q)}{\partial Q_i}\frac{\partial Q_k (q,p)}{\partial q_j}= \frac{\partial P_i (q,Q)}{\partial q_j}-\Sigma_{k,l} \frac{\partial P_k (q,Q)}{\partial q_l}\frac{\partial q_l (Q,P)}{\partial Q_k}\frac{\partial Q_k (q,p)}{\partial q_j}= \frac{\partial P_i (q,Q)}{\partial q_j}+\Sigma_{k,l} \frac{\partial p_l (q,Q)}{\partial Q_k}\frac{\partial q_l (Q,P)}{\partial Q_k}\frac{\partial Q_k (q,p)}{\partial q_j}$  まで計算したところで八方ふさがりになりました。 -- [[吉川晃生]] &new{2021-09-18 (土) 15:40:21};
- まず、最後の式は$= \frac{\partial P_i (q,Q)}{\partial q_j}+\Sigma_{k,l} \frac{\partial p_l (q,Q)}{\partial Q_k}\frac{\partial q_l (Q,P)}{\partial Q_i}\frac{\partial Q_k (q,p)}{\partial q_j}$ですね。 -- [[前野]] &new{2021-09-19 (日) 07:53:18};
- ここで(D.21)から$-  {\partial p_\ell(q,Q)\over \partial q_j}   =\sum_k{\partial p_\ell(q,Q)\over \partial Q_k}{\partial Q_k(q,p)\over \partial q_j}$が言えます。 -- [[前野]] &new{2021-09-19 (日) 08:00:26};
- これでまず、$=\frac{\partial P_i (q,Q)}{\partial q_j}-\Sigma_{l} \frac{\partial p_l (q,Q)}{\partial q_j}\frac{\partial q_l (Q,P)}{\partial Q_i}$になります。 -- [[前野]] &new{2021-09-19 (日) 08:03:24};
- 次に(D.19)から${\partial p_l(q,Q)\over\partial q_i}={\partial p_i(q,Q)\over\partial q_l}$です。もうひとつ、 (D.18)から${\partial P_i(q,Q)\over\partial q_j}=-{\partial p_j(q,Q)\over\partial Q_i}$を使うと、$=-{\partial p_j(q,Q)\over\partial Q_i}-\sum_l{\partial p_j(q,Q)\over\partial q_l}{\partial q_l(Q,P)\over\partial Q_i}$となります。 -- [[前野]] &new{2021-09-19 (日) 08:10:48};
- これは$=-{\partial p_j(P,Q)\over\partial Q_i}$です。使っている式は(D.18)、(D.19)、(D.21)です。第1項にも(D.18)が使えるところがわかりにくかったでしょうか。 -- [[前野]] &new{2021-09-19 (日) 08:13:01};

#comment

**問い10-5 [#w59358f6]
>[[吉川晃生]] (2021-09-14 (火) 22:18:55)~
~
$\frac{\partial P}{\partial p} = \frac{\partial q}{\partial Q}, ~
\frac{\partial Q}{\partial p} = -\frac{\partial q}{\partial P}$~
を示せれば$J=1$となるのはどうしてですか?~

//
- (10.5)のJの式にそれを代入してみてください。「qをqで微分する」式になります。 -- [[前野]] &new{2021-09-14 (火) 23:22:37};
- 自分の本を読み直してみたら、(10.6)を使えばすぐにわかりますね。 -- [[前野]] &new{2021-09-15 (水) 11:55:45};
- 理解できました。ありがとうございました。 -- [[吉川晃生]] &new{2021-09-17 (金) 22:01:20};

#comment

**P.38 FAQについて [#l5722663]
>[[michi]] (2021-09-08 (水) 23:03:32)~
~
$\int(なにか)\delta y(x) dx =0$について,なぜ$\delta y(x)$が独立でなければ$(なにか)=0$とすることができないのか,詳しく教えていただけないでしょうか.~

//
- 例をあげますと。$a$と$b$が独立ならば「$ax+by=0$なら$x=0$かつ$y=0$」といえます。しかし、$a=b$という関係がある場合、$ax+by=0$からは$x+y=0$しか言えません。 -- [[前野]] &new{2021-09-09 (木) 07:26:28};

#comment

**無題 [#n0b1c357]
>[[yamashita]] (2021-05-18 (火) 20:15:44)~
~
先生に質問して良いのか、迷ったのですが、質問させてください。~
(教科書には載っていない内容なので迷いました。)~
磁場がある場合の荷電粒子のラグランジアンで共役運動量を求めると、p=mv+eAとなりますが、~
このeAの部分には何か物理的な意味合いがあるのでしょうか?~
それとも解析力学で形式上出てきた項なのでしょうか?~

//
- もちろん、共役運動量がデカルト座標の運動量と違うことが多々あるのは知っているのですが、項ごとに物理的な意味合いがあるのでしょうか? -- [[yamashita]] &new{2021-05-18 (火) 20:18:24};
- 物理的意味があるに決まってます。「解析力学で形式上出てきた項」だからこそ、物理的意味があります(この二つは排他的なものではない)。ベクトルポテンシャルがどういう意味を持っているかは電磁気の本を読めばよいと思いますが。 -- [[前野]] &new{2021-05-18 (火) 21:08:41};
- そもそも運動方程式を作ったときに磁場の影響が出てくるように導入しているものなのだから、物理的意味はそこにあります。 -- [[前野]] &new{2021-05-18 (火) 21:09:36};
- ありがとうございます。「運動方程式に磁場の影響が出てくるように導入した」という意味で、自然界に存在する量(?←この書き方もどうかと思いますが)ではない(かも知れない)ということでしょうか? -- [[yamashita]] &new{2021-05-19 (水) 05:22:14};
- ?? 「運動方程式に出てくるんだから自然界に存在しているでしょ」と上で言ったつもりなんですが。 -- [[前野]] &new{2021-05-19 (水) 05:43:10};
- 要領を得ず、すみません。p=mv+eAのmvは電子の運動量、では残りのeAは何なのでしょうか?結局、共役運動量が何なのかが分からないでいるのです。 -- [[yamashita]] &new{2021-05-19 (水) 21:41:12};
- では逆に、あなたの言う『運動量」って何なのです?? 解析力学での運動量の定義は${\partial L\over\partial \dot x}$だったり、${\partial S\over \partial x}$だったりするものですから、mv+eAこそが運動量です。「運動量はmv」という考えは解析力学では違います。 -- [[前野]] &new{2021-05-19 (水) 22:04:29};
- 「運動量はmvであって欲しい」ということなら、解析力学での運動量はそうではなく、もっと広い概念です。 -- [[前野]] &new{2021-05-19 (水) 22:05:05};
- ありがとうございます。『mv+eAこそが運動量です。「運動量はmv」という考えは解析力学では違います』や『解析力学での運動量はそうではなく、もっと広い概念です』とある様に、解析力学の中の概念だ、ということなのですね。(結局やはり共役運動量が分かってないわけですが。) -- [[yamashita]] &new{2021-05-19 (水) 22:27:04};
- 初等力学(?)に焼き直して考えられないものか、と考えてしまうので、勉強します。 -- [[yamashita]] &new{2021-05-19 (水) 22:29:24};

#comment

**p.158式(6.70)及びp.162下2行 [#j3804f94]
>[[Inaba]] (2021-04-24 (土) 21:39:19)~
~
・式(6.70)で$y_n=$と置いてますが、この式の右辺は行列Kの固有ベクトルの成分であって、n番目の質点の変位ではないと思うのですが、どういうことなのでしょうか?~
・p.162の下2行で、「この解の一個一個のモードをみると〜」とあるのですが、「この解」が示されてはいません。ここで示されているのは極限をとった結果の角振動数だけです。単純な問題ではあるので、解の形を求めるのは省略して結果だけ書いたということでしょうか?~

//
- 式(6.70)の$y_n$は「固有ベクトルの成分」です。そしてそれは「その固有ベクトルで表される振動をしているときの、n番目の質点の変位」に比例してます(一般的な振動の変位は、いろんなpの$y_n$の線形結合になってます)。 -- [[前野]] &new{2021-04-25 (日) 00:15:12};
- p162については、三角関数で表されることはわかっているので、振動数がわかれば「この解」の中身はもうわかっているでしょ、というつもりです。 -- [[前野]] &new{2021-04-25 (日) 00:18:02};
- ありがとうございます。 -- [[Inaba]] &new{2021-04-25 (日) 12:19:52};
- ありがとうございます。 -- [[Inaba]] &new{2021-04-25 (日) 12:19:53};
- 固有ベクトルの成分がn番目の質点の変位に比例するというのは{\bf y}={\bf TY}から言えることでしょうか?あくまでも$y_n$は{\bf y}の成分の一つで、固有ベクトルの成分は{\bf T}を構成する固有ベクトルの成分のひとつであるというのが私の認識で、「比例する」も「$y_n$=固有ベクトルの成分と置くこと」もピンと来ないです...。(ベクトルは太字で書かせていただきました) -- [[Inaba]] &new{2021-04-25 (日) 12:44:51};
- すみません、太字をうまく書けませんでした。{\bf 〇}としたものが太字です。 -- [[Inaba]] &new{2021-04-25 (日) 12:47:02};
- (6.70)のあたりで計算していることは、Lの中の位置エネルギーの部分に入っている行列${\bf K}$の固有ベクトルを求めようとして、$y_n$のそれぞれに「固有ベクトルの候補」となる$\sqrt{2\over N+1}\sin{np\pi\over N+1}$を代入してみましょう、ということです。 -- [[前野]] &new{2021-04-25 (日) 13:49:04};
- つまり、(6.70)を書き下した時点では、右辺は「n番目の質点の変位がこうなってくれたらこのベクトルは${\bf K}$の固有ベクトルになってくれるんじゃないかな?」という「予想」の式です(その予想はあたっていることがそのあとでわかるわけですから)。 -- [[前野]] &new{2021-04-25 (日) 13:50:33};
- そういう予想なので、$y_n$が「n番目の質点の変位」なのは元々そうだからで、「固有ベクトルの成分」なのは「これからそうなることを証明しよう」ということです。 -- [[前野]] &new{2021-04-25 (日) 13:51:27};
- つまりは(6.69)までの$y_n$は一般的な変位で、(6.70)は「もしも変位がこうなってたら、それは${\bf K}$の固有ベクトルになっている(だろう)」という条件つきの$y_n$です。 -- [[前野]] &new{2021-04-25 (日) 13:52:38};

#comment

**p.155変数変換 [#h00eadfa]
>[[きょんきょん]] (2021-04-07 (水) 17:43:00)~
~
155ページの変数変換は小文字と大文字を入れ替えて「$\left[\begin{array}{c}x_{1}\\x_{2}\\x_{3}\\\end{array}\right]=\mathbf{T}\left[\begin{array}{c}X_{1}\\X_{2}\\X_{3}\\\end{array}\right]$となるように新しい変数を導入してラグランジアンを書き直すと、…」としたほうがいいかもしれません。そうしないと$\mathbf{T}$が対称行列でないときにうまくいかないことがあります。~

//
- あ、すみません。確かにこれは、固有ベクトルを並べた行列にXの方を書けるようにしないとだめですね。 -- [[前野]] &new{2021-04-07 (水) 20:04:45};
- 「(新しく導入される記号)=(なにか)」の形のほうがきれいなので$\begin{pmatrix}X_{1}&X_{2}&X_{3}\end{pmatrix}=\begin{pmatrix}x_{1}&x_{2}&x_{3}\end{pmatrix}\mathbf{T}$のほうがいいかもしれません。 -- [[きょんきょん]] &new{2021-04-07 (水) 20:10:51};
- 20時10分の投稿は撤回します。 -- [[きょんきょん]] &new{2021-04-07 (水) 20:43:26};

#comment



#hr
これより古い記事は

-[[「よくわかる解析力学」サポート掲示板(2021年3月まで)]]
-[[「よくわかる解析力学」サポート掲示板(2020年10月まで)]]
-[[「よくわかる解析力学」サポート掲示板(2019年12月まで)]]
-[[「よくわかる解析力学」サポート掲示板(2018年12月まで)]]
-[[「よくわかる解析力学」サポート掲示板(2018年7月まで)]]
-[[「よくわかる解析力学」サポート掲示板(2018年1月まで)]]
-[[「よくわかる解析力学」サポート掲示板(2017年4月まで)]]
-[[「よくわかる解析力学」サポート掲示板(2017年3月まで)]]
-[[「よくわかる解析力学」サポート掲示板(2016年12月まで)]]
-[[「よくわかる解析力学」サポート掲示板(2016年7月まで)]]
-[[「よくわかる解析力学」サポート掲示板(2016年5月まで)]]
-[[「よくわかる解析力学」サポート掲示板(2015年9月)]]
-[[「よくわかる解析力学」サポート掲示板(2015年7月〜8月)]]
-[[「よくわかる解析力学」サポート掲示板(2015年前半)]]
-[[よくわかる解析力学サポート掲示板(2014年まで)]]
にあります。

トップ   新規 一覧 単語検索 最終更新   ヘルプ   最終更新のRSS